Can W be at 11 and Y be at 2:00?
Is the rule stating -WY mean they both need to be in the morning or both in the afternoon? Or is ...
hatemz on June 23, 2020
  • December 2015 LSAT
  • SEC3
  • Q1
2
Replies
Can someone please explain this?
Please explain why B is correct.
hkolon on June 23, 2020
  • December 2015 LSAT
  • SEC3
  • Q21
1
Reply
Answer B
Please expain why B is correct.
saskipper on June 23, 2020
  • December 2015 LSAT
  • SEC3
  • Q21
1
Reply
Why B?
Hi- Can you please explain why B is the correct answer? According to rule 2, N can only be placed...
josettebrooksbank on June 12, 2020
  • December 2015 LSAT
  • SEC3
  • Q9
2
Replies
Help
I do not know how to set this game up and the explanation is no longer on here.
shylahmarieg@gmail.com on May 31, 2020
  • December 2015 LSAT
  • SEC3
  • Q13
1
Reply
Can't eliminate between b and e. What am I miss...
Thanks! Is there a deduction video for this one?
ashwinid on February 19, 2020
  • December 2015 LSAT
  • SEC3
  • Q2
1
Reply
Error
I can't see any of the options after the question. No A,B,C,D
yesenialva03 on January 28, 2020
  • December 2015 LSAT
  • SEC3
  • Q1
1
Reply
any guidance on how to arrive at the correct an...
kindly show me how to arrive at the correct answer for this question
Tony on September 1, 2019
  • December 2015 LSAT
  • SEC3
  • Q23
1
Reply
Can you please explain how we should know that ...
Thank you
Bryan on June 8, 2019
  • December 2015 LSAT
  • SEC3
  • Q11
1
Reply
Can you please help with the set up of this game?
I do not know what inferences to make here
Bryan on June 8, 2019
  • December 2015 LSAT
  • SEC3
  • Q19
1
Reply
Video
The game set up video is not working
Bryan on June 8, 2019
  • December 2015 LSAT
  • SEC3
  • Q13
3
Replies
How is B correct?
How is B the correct answer? I understand the set up but I am not understanding how B is right. T...
Anna on November 14, 2018
  • December 2015 LSAT
  • SEC3
  • Q13
1
Reply
Game setup
Can you please post a set up for this game
Kuma on September 7, 2018
  • December 2015 LSAT
  • SEC3
  • Q17
2
Replies